A query regarding Rotational Invariance

In summary: It looks like the probabilities in #9 are for measuring spin up or down in the direction defined by the angles ##\theta, \phi##, denoted by ##n##. These are not probabilities for different observables, just two different outcomes for the same observable (namely spin in the direction ##n##) depending on the state of the system (namely ##\psi##).The probabilities in #9 do add to 1. I used the fact that ##|\alpha|^2 + |\beta|^2 = 1##. You can see that in the equations I wrote in #11. Also, in the second line of #9, I wrote down the equations for the probabilities of measuring spin up
  • #1
Student149
58
0
We know that Bell States follow the Rotational Invariance property i.e. the probability of results on measurement of two entangled particles do not change if the initial measurement basis (say ##u##) is rotated by an angle θ to a new basis (to say ##v##).

Lets take the Bell State ##\psi = \frac 1 {\sqrt 2}(\uparrow \uparrow + \downarrow \downarrow)##. Here both the terms have the probability ##50##% in basis ##u## or rotated basis ##v##.

How does the Rotational Invariance property get affected if the probability of two terms is unequal i.e. if the entangled state is of the form ##\psi = (α\uparrow \uparrow + β\downarrow \downarrow)##, ##α^2+β^2=1##, ##α≠β##?

My guess is they remain unchanged as it is a generic case of Bell States.
 
Last edited:
Physics news on Phys.org
  • #2
How well does your guess work for the extreme case ##\alpha=0## and ##\beta=1##?
 
  • #3
Nugatory said:
How well does your guess work for the extreme case ##\alpha=0## and ##\beta=1##?

haha. That is why it was a guess. I stated that assuming non zero real values. I am not sure how to compute the generic case hence the Q?
 
  • #4
Student149 said:
We know that Bell States follow the Rotational Invariance property i.e. the probability of results on measurement of two entangled particles do not change if the initial measurement basis (say ##u##) is rotated by an angle θ to a new basis (to say ##v##).

Lets take the Bell State ##\psi = \frac 1 {\sqrt 2}(\uparrow \uparrow + \downarrow \downarrow)##. Here both the terms have the probability ##50##% in basis ##u## or rotated basis ##v##.

How does the Rotational Invariance property get affected if the probability of two terms is unequal i.e. if the entangled state is of the form ##\psi = (α\uparrow \uparrow + β\downarrow \downarrow)##, ##α^2+β^2=1##, ##α≠β##?

My guess is they remain unchanged as it is a generic case of Bell States.
You shouldn't be guessing, you should be calculating! I make it that the probability of getting spin up in a direction making angle ##\theta## with the z-axis is:
$$\beta^2 + (1 - 2\beta^2)\cos^2(\frac \theta 2)$$
You might want to double check that. It works out for ##\beta = \frac 1 {\sqrt 2}##.
 
  • Like
Likes vanhees71 and Student149
  • #5
Student149 said:
haha. That is why it was a guess. I stated that assuming non zero real values. I am not sure how to compute the generic case hence the Q?
I can post the details if you are interested.
 
  • #6
PeroK said:
I can post the details if you are interested.

Thank you. I would be thankful..
 
  • #7
Student149 said:
Thank you. I would be thankful..
The key is to work out the ##\uparrow_z## and ##\downarrow_z## states in an arbitrary basis for a measurement direction defined by spherical angles ##\theta, \phi## and denoted by ##n##. These are:
$$\uparrow_z = \cos(\frac \theta 2)\uparrow_n - \sin(\frac \theta 2)\downarrow_n$$ and
$$\downarrow_z = e^{-i\phi}(\sin(\frac \theta 2)\uparrow_n + \cos(\frac \theta 2)\downarrow_n)$$
You can plug these into the required Bell state and expand. For the state you used in the OP, I get:
$$\psi = [\alpha \cos^2(\frac \theta 2) + \beta e^{-2i\phi}\sin^2(\frac \theta 2)]\uparrow_n\uparrow_n + [\alpha \sin^2(\frac \theta 2) + \beta e^{-2i\phi}\cos^2(\frac \theta 2)]\downarrow_n\downarrow_n + \\ \frac 1 2\sin\theta(\beta e^{-2i\phi} - \alpha)(\uparrow_n\downarrow_n + \downarrow_n\uparrow_n)$$
It would be good if you could double check that, as well as the probability calculations from that :smile:
 
  • Like
Likes vanhees71 and Student149
  • #8
PeroK said:
You shouldn't be guessing, you should be calculating! I make it that the probability of getting spin up in a direction making angle θ with the z-axis is:
β2+(1−2β2)cos2⁡(θ2)
You might want to double check that. It works out for β=12.
PeroK said:
The key is to work out the ##\uparrow_z## and ##\downarrow_z## states in an arbitrary basis for a measurement direction defined by spherical angles ##\theta, \phi## and denoted by ##n##. These are:
$$\uparrow_z = \cos(\frac \theta 2)\uparrow_n - \sin(\frac \theta 2)\downarrow_n$$ and
$$\downarrow_z = e^{-i\phi}(\sin(\frac \theta 2)\uparrow_n + \cos(\frac \theta 2)\downarrow_n)$$
You can plug these into the required Bell state and expand. For the state you used in the OP, I get:
$$\psi = [\alpha \cos^2(\frac \theta 2) + \beta e^{-2i\phi}\sin^2(\frac \theta 2)]\uparrow_n\uparrow_n + [\alpha \sin^2(\frac \theta 2) + \beta e^{-2i\phi}\cos^2(\frac \theta 2)]\downarrow_n\downarrow_n + \\ \frac 1 2\sin\theta(\beta e^{-2i\phi} - \alpha)(\uparrow_n\downarrow_n + \downarrow_n\uparrow_n)$$
It would be good if you could double check that, as well as the probability calculations from that :smile:

I would. much appreciated.
 
  • #9
Student149 said:
I would. much appreciated.
It all seems to check out. I can simplify the equations a bit:
$$p(\uparrow_n | \psi) = \frac 1 2 + (\frac 1 2 - |\beta|^2)\cos \theta = \frac 1 2 - (\frac 1 2 - |\alpha|^2)\cos \theta$$ $$p(\downarrow_n | \psi) = \frac 1 2 + (\frac 1 2 - |\alpha|^2)\cos \theta = \frac 1 2 - (\frac 1 2 - |\beta|^2)\cos \theta$$
Note that unless ##|\alpha^2| = |\beta^2| = \frac 1 2## these probabilities vary with ##\theta##. Note also that if ##\theta = \frac \pi 2## (i.e. we are measuring in the x-y plane), then the probabilities are ##1/2## independent of ##\alpha, \beta##.
 
  • Like
Likes Student149
  • #10
PS physically we should have expected the probabilities to depend on ##\theta## but not on ##\phi##. I could have set ##\phi = 0## and made things easier.
 
  • #11
Why don't the probs. add to 1? Also the very idea in #1 seems to be flawed to me. This is a state in the triplet, i.e., for ##S=1## and thus not rotationally invariant.
 
  • #12
vanhees71 said:
Why don't the probs. add to 1? Also the very idea in #1 seems to be flawed to me. This is a state in the triplet, i.e., for ##S=1## and thus not rotationally invariant.

I am confused. I think they do add to 1 in #9. Please elaborate what is flawed in #1 (I am a novice in QM, but I can't find a flaw)
 
  • #13
The two-spin state in #1 is
$$|\Psi \rangle=\frac{1}{\sqrt{2}} (|1/2,1/2 \rangle + |-1/2,-1/2 \rangle).$$
This is a state belonging to total spin ##S=1##, i.e., the spin-triplet state and thus is not rotationally invariant.

I also do not understand the notation in #9. The probabilities for the measurement of which observable are stated there? Also the numbers don't add to 1 but to ##1/2(1-\cos \theta)+(|\alpha|^2+|\beta|^2)\cos \theta=1/2(1+\cos \theta)##.
 
  • #14
1. I picked my example from here:

2. The ##- (\frac 1 2 )\cos \theta## is twice. Hence, it cancels ##+\cos \theta##

3. If the direction of measurement is at an angle ##\theta## w.r.t. the original direction vector the probability of spin up or down in that new direction (it can be alternatively considered as the basis rotation w.r.t the original basis if i understand correctly)..
 
  • #15
vanhees71 said:
The two-spin state in #1 is
$$|\Psi \rangle=\frac{1}{\sqrt{2}} (|1/2,1/2 \rangle + |-1/2,-1/2 \rangle).$$
This is a state belonging to total spin ##S=1##, i.e., the spin-triplet state and thus is not rotationally invariant.

I also do not understand the notation in #9. The probabilities for the measurement of which observable are stated there? Also the numbers don't add to 1 but to ##1/2(1-\cos \theta)+(|\alpha|^2+|\beta|^2)\cos \theta=1/2(1+\cos \theta)##.
The probabilities in post #9 fairly obviously add to ##1##.
 
  • #16
vanhees71 said:
I also do not understand the notation in #9.
##p(\uparrow_n | \psi)## is the probablity of getting spin up in direction ##\vec n##, at an angle of ##\theta## to the z-axis, given the state ##\psi## in the OP.
 
  • #17
PeroK said:
The probabilities in post #9 fairly obviously add to ##1##.
Ok, then I don't understand the notation.
 
  • #18
PeroK said:
##p(\uparrow_n | \psi)## is the probablity of getting spin up in direction ##\vec n##, at an angle of ##\theta## to the z-axis, given the state ##\psi## in the OP.
There are two particles. Do you mean the spin of one of the particles or the total spin? It's hard to discuss, if the expressions are not well defined.

Let's do a well-defined example. I suppose that ##|\pm 1/2 \rangle## are the usual eigenstates of ##\hat{\sigma}_z## with eigenvalues ##\pm 1/2## (##\hbar=1##).

The two-spin state is ##|\Psi \rangle \langle \Psi|## with
$$|\Psi \rangle=\alpha |1/2,1/2 \rangle + \beta |-1/2,-1/2 \rangle, \quad |\alpha|^2+|\beta|^2$$
Now let's measure a spin component in direction taking an angle ##\theta## with the 3-direction. To be concrete, consider ##\phi=0## in the usual spherical coordinates. Then
$$\hat{\sigma}_{\theta}=\sin \theta \hat{\sigma}_x + \cos \theta \hat{\sigma}_z.$$
The eigenvectors are
$$|\chi_+ \rangle=\cos(\theta/2) |1/2 \rangle + \sin(\theta/2) |-1/2 \rangle$$
and
$$|\chi_- \rangle = -\sin(\theta/2) |1/2 \rangle + \cos(theta/2)|-1/2 \rangle.$$
The probability to measure ##+1/2## for the 1st spin thus is
$$P_+=\sum_{j =\pm 1/2} |\langle \chi_+,j|\Psi \rangle|^2.$$
Now
$$\langle \chi_+,1/2|\Psi \rangle=\alpha \cos(\theta/2), \quad \langle \chi_+,-1/2|\Psi \rangle=\beta \sin(\theta/2)$$
and thus
$$P_+=|\alpha|^2 \cos^2(\theta/2)+|\beta|^2 \sin^2 (\theta/2).$$
For ##P_-## we need
$$\langle \chi_-,1/2|\Psi \rangle=-\alpha \sin(\theta/2), \quad \langle \chi_-,-1/2 \rangle=\beta \cos(\theta/2)$$
and thus
$$P_-=|\alpha^2| \sin^2(\theta/2)+|\beta|^2 \cos^2(\theta/2).$$
I hope, I've not made a mistake, but at least now the probabilities clearly add to 1 :-)).
 
Last edited:
  • #19
vanhees71 said:
There are two particles. Do you mean the spin of one of the particles or the total spin? It's hard to discuss, if the expressions are not well defined.
Yes, sorry, the spin of one particle.
 
  • Like
Likes vanhees71
  • #20
vanhees71 said:
There are two particles. Do you mean the spin of one of the particles or the total spin? It's hard to discuss, if the expressions are not well defined.

Let's do a well-defined example. I suppose that ##|\pm 1/2 \rangle## are the usual eigenstates of ##\hat{\sigma}_z## with eigenvalues ##\pm 1/2## (##\hbar=1##).

The two-spin state is ##|\Psi \rangle \langle \Psi|## with
$$|\Psi \rangle=\alpha |1/2,1/2 \rangle + \beta |-1/2,-1/2 \rangle, \quad |\alpha|^2+|\beta|^2$$
Now let's measure a spin component in direction taking an angle ##\theta## with the 3-direction. To be concrete, consider ##\phi=0## in the usual spherical coordinates. Then
$$\hat{\sigma}_{\theta}=\sin \theta \hat{\sigma}_x + \cos \theta \hat{\sigma}_z.$$
The eigenvectors are
$$|\chi_+ \rangle=\cos(\theta/2) |1/2 \rangle + \sin(\theta/2) |-1/2 \rangle$$
and
$$|\chi_- \rangle = -\sin(\theta/2) |1/2 \rangle + \cos(theta/2)|-1/2 \rangle.$$
The probability to measure ##+1/2## for the 1st spin thus is
$$P_+=\sum_{j =\pm 1/2} |\langle \chi_+,j|\Psi \rangle|^2.$$
Now
$$\langle \chi_+,1/2|\Psi \rangle=\alpha \cos(\theta/2), \quad \langle \chi_+,-1/2|\Psi \rangle=\beta \sin(\theta/2)$$
and thus
$$P_+=|\alpha|^2 \cos^2(\theta/2)+|\beta|^2 \sin^2 (\theta/2).$$
For ##P_-## we need
$$\langle \chi_-,1/2|\Psi \rangle=-\alpha \sin(\theta/2), \quad \langle \chi_-,-1/2 \rangle=\beta \cos(\theta/2)$$
and thus
$$P_-=|\alpha^2| \sin^2(\theta/2)+|\beta|^2 \cos^2(\theta/2).$$
I hope, I've not made a mistake, but at least now the probabilities clearly add to 1 :-)).

So, just to be clear/reconfirm:

if Alice and Bob have 1 particle each and Alice decides to measure its particle (in a direction that has an angle ##\theta## with the original direction ##z##) the probability of both Alice and Bob getting a ##\uparrow \uparrow## and getting a ##\downarrow \downarrow## in new direction is given by the eqns in #9 correctly. right?
 
  • #21
Student149 said:
So, just to be clear/reconfirm:

if Alice and Bob have 1 particle each and Alice decides to measure its particle (in a direction that has an angle ##\theta## with the original direction ##z##) the probability of both Alice and Bob getting a ##\uparrow \uparrow## and getting a ##\downarrow \downarrow## in new direction is given by the eqns in #9 correctly. right?
No. Post #9 was for either Alice's particle or Bob's particle.

The probability that the spins are opposite in this direction for the Bell state ##\frac 1 {\sqrt 2}(\uparrow \uparrow + \downarrow \downarrow)## is:
$$\frac 1 2 \sin^2 \theta (1 - \cos (2\phi))$$
This shows that the state is not as rotationally invariant as you may have believed. In fact, in the y-direction, with ##\theta = \phi = \frac \pi 2##, we have total anticorrelation:
$$\frac 1 {\sqrt 2}(\uparrow_z \uparrow_z + \downarrow_z \downarrow_z) = -\frac 1 {\sqrt 2}(\uparrow_y \downarrow_y + \downarrow_y \uparrow_y)$$
 
  • #22
PeroK said:
No. Post #9 was for either Alice's particle or Bob's particle.

The probability that the spins are opposite in this direction for the Bell state ##\frac 1 {\sqrt 2}(\uparrow \uparrow + \downarrow \downarrow)## is:
$$\frac 1 2 \sin^2 \theta (1 - \cos (2\phi))$$
This shows that the state is not as rotationally invariant as you may have believed. In fact, in the y-direction, with ##\theta = \phi = \frac \pi 2##, we have total anticorrelation:
$$\frac 1 {\sqrt 2}(\uparrow_z \uparrow_z + \downarrow_z \downarrow_z) = -\frac 1 {\sqrt 2}(\uparrow_y \downarrow_y + \downarrow_y \uparrow_y)$$

Now I am terribly lost to the point of giving up any hope of understanding even the basics unless I have a complete formal background in physics. The original Q was regarding arbitrary ##α, β## and I assumed there is only one angle ##θ## (as described in the video). I have no clue about the other details (my background is CS), namely ##\phi## etc. Thank you for the help though..
 
  • #23
Student149 said:
Now I am terribly lost to the point of giving up any hope of understanding even the basics unless I have a complete formal background in physics. The original Q was regarding arbitrary ##α, β## and I assumed there is only one angle ##θ## (as described in the video). I have no clue about the other details (my background is CS), namely ##\phi## etc. Thank you for the help though..
The video considers rotation in a plane (defined by a single angle ##\theta##). The state has rotational invariance in a plane. But, nature is three dimensional (hence rotations in 3D are defined by two angles ##\theta## and ##\phi##). That Bell state does not have 3D rotational invariance.

And, in answer to your question, you lose all rotational invariance unless you have ##|\alpha| = |\beta| = \frac 1 {\sqrt 2}##.
 
  • #24
PS the Bell state ##\frac 1 {\sqrt 2}(\uparrow \downarrow - \downarrow \uparrow)##, which represents the spin-0 singlet state, has full 3D rotational invariance - which the other Bell states do not.
 
  • #25
PeroK said:
PS the Bell state ##\frac 1 {\sqrt 2}(\uparrow \downarrow - \downarrow \uparrow)##, which represents the spin-0 singlet state, has full 3D rotational invariance - which the other Bell states do not.

Got it. But if we limit ourselves to a plane (which I was assuming as its in the video example hence the confusion) we can set ##\phi##=0 in #21 then from it we can get the probability of opposite spin. But, it doesn't answer about the probability of ##\uparrow \uparrow## and ##\downarrow \downarrow## each given arbitrary α, β.

PS Sorry for testing your patience.
 
  • #26
Student149 said:
Got it. But if we limit ourselves to a plane (which I was assuming as its in the video example hence the confusion) we can set ##\phi##=0 in #21 then from it we can get the probability of opposite spin. But, it doesn't answer about the probability of ##\uparrow \uparrow## and ##\downarrow \downarrow## each given arbitrary α, β.

PS Sorry for testing your patience.
With ##\phi = 0##, the probability of getting opposite spins is:
$$\frac 1 2 |\alpha - \beta|^2 \sin^2 \theta $$
 
  • #27
PeroK said:
With ##\phi = 0##, the probability of getting opposite spins is:
$$\frac 1 2 |\alpha - \beta|^2 \sin^2 \theta $$

I totally understand.

But how do we know about the same spin probability of each case ↑↑ and ↓↓ given arbitrary α, β?
 
  • #28
Student149 said:
I totally understand.

But how do we know about the same spin probability of each case ↑↑ and ↓↓ given arbitrary α, β?
You use the coefficients of the state expressed in the relevant basis. From a few posts ago:
$$\psi = [\alpha \cos^2(\frac \theta 2) + \beta e^{-2i\phi}\sin^2(\frac \theta 2)]\uparrow_n\uparrow_n + [\alpha \sin^2(\frac \theta 2) + \beta e^{-2i\phi}\cos^2(\frac \theta 2)]\downarrow_n\downarrow_n$$ $$ + \frac 1 2\sin\theta(\beta e^{-2i\phi} - \alpha)(\uparrow_n\downarrow_n + \downarrow_n\uparrow_n)$$
 
  • Like
Likes Student149
  • #29
PeroK said:
You use the coefficients of the state expressed in the relevant basis. From a few posts ago:
$$\psi = [\alpha \cos^2(\frac \theta 2) + \beta e^{-2i\phi}\sin^2(\frac \theta 2)]\uparrow_n\uparrow_n + [\alpha \sin^2(\frac \theta 2) + \beta e^{-2i\phi}\cos^2(\frac \theta 2)]\downarrow_n\downarrow_n$$ $$ + \frac 1 2\sin\theta(\beta e^{-2i\phi} - \alpha)(\uparrow_n\downarrow_n + \downarrow_n\uparrow_n)$$

This has been very helpful. Thanks a ton again.
 
  • #30
Student149 said:
So, just to be clear/reconfirm:

if Alice and Bob have 1 particle each and Alice decides to measure its particle (in a direction that has an angle ##\theta## with the original direction ##z##) the probability of both Alice and Bob getting a ##\uparrow \uparrow## and getting a ##\downarrow \downarrow## in new direction is given by the eqns in #9 correctly. right?
Ok, that's a different question.

To answer the question about joint measurements when both are measuring the spin component in direction ##\theta## (using the same states as in my previous posting) it's most simple to first express the old basis by the new, i.e.,
$$|1/2 \rangle=\cos(\theta/2) |\chi_+ \rangle - \sin[\theta/2]|\chi_- \rangle, \quad |-1/2 \rangle = \sin(\theta/2) |1/2 \rangle + \cos(\theta/2) |\chi_- \rangle.$$
The state ket the particle is prepared in is thus given in this basis by
$$|\Psi \rangle = [\alpha \cos^2(\theta/2) + \beta \sin^2(\theta/2)] |\chi_+,\chi_+ \rangle + (\beta-\alpha) \cos(\theta/2) \sin(\theta/2) (|1/2,-1/2 \rangle+|-1/2,1/2 \rangle) + (\alpha \sin^2 \theta/2 + \beta \cos^2 \theta/2) |-1/2,-1/2 \rangle.$$
From this you read off immediately for the probabilities for the 4 possible outcomes of joint measurements of the spin components in ##\theta## direction
$$P_{++}=|\alpha \cos^2 \theta/2 + \beta \sin^2 \theta/2|^2,$$
$$P_{+-}=P_{-+}=|\alpha-\beta|^2 \sin^2 \theta/2 \cos^2 \theta/2,$$
$$P_{--}=|\alpha \sin^2 \theta/2+\beta \cos^2 \theta/2)|^2.$$
You can simplify a bit using some trigonometry:
$$P_{++}=\frac{1}{4} [|\alpha+\beta|^2 + 2 (|\alpha|^2-|\beta|^2)\cos \theta + |\alpha-\beta|^2 \cos^2 \theta],$$
$$P_{+-}=P_{-+}=\frac{1}{4} |\alpha-\beta|^2 \sin^2 \theta,$$
$$P_{--}=\frac{1}{4}[|\alpha+\beta|^2 - 2 (|\alpha|^2-|\beta|^2) \cos \theta + |\alpha-\beta|^2 \cos^2 \theta].$$
As expected, the probabilities in general depend on ##\theta##. That's because any spin-triplet state is not rotation invariant, because the total spin is ##S=+1##. The spin-singlet state, i.e., ##S=0## is the only rotationally invariant state, i.e.,
$$|\Psi ' \rangle=\frac{1}{\sqrt{2}}(|1/2,-1/2 \rangle-|-1/2,1/2 \rangle),$$
as already mentioned in some postings above.
 
Last edited:
  • Like
Likes Student149 and PeroK

1. What is rotational invariance?

Rotational invariance is a property that describes an object or system that remains unchanged under rotations. This means that the object or system will look the same regardless of its orientation or angle of rotation.

2. Why is rotational invariance important in science?

Rotational invariance is important in science because it allows us to simplify and generalize our understanding of physical phenomena. By considering rotational invariance, we can make predictions and draw conclusions that are applicable in different orientations or angles of rotation.

3. How is rotational invariance applied in different scientific fields?

Rotational invariance is applied in various scientific fields, such as physics, chemistry, and biology. In physics, rotational invariance is used to describe the laws of motion and conservation of angular momentum. In chemistry, it is used to explain molecular structures and symmetry. In biology, it is used to study the structure and function of biological molecules and organisms.

4. Can rotational invariance be violated?

Yes, rotational invariance can be violated in certain situations. This typically occurs when there is an external force or influence acting on the object or system, causing it to change its orientation or angle of rotation.

5. How is rotational invariance related to symmetry?

Rotational invariance and symmetry are closely related. Both concepts involve the idea of an object or system remaining unchanged under certain transformations. In rotational invariance, the transformation is a rotation, while in symmetry, it can be any type of transformation, such as translation or reflection.

Similar threads

Replies
12
Views
766
Replies
8
Views
753
Replies
1
Views
642
  • Quantum Physics
Replies
12
Views
1K
  • Quantum Interpretations and Foundations
2
Replies
47
Views
1K
  • Advanced Physics Homework Help
Replies
3
Views
2K
  • Quantum Physics
3
Replies
87
Views
5K
  • Quantum Physics
Replies
1
Views
936
  • Quantum Physics
Replies
3
Views
7K
Replies
2
Views
714
Back
Top